Write the equation of the line which passes through (5, −2) and (3, 4) in standard form.

Answers

Answer 1

The equation of the line is 3x + y = 1

How to determine the equation of the line?

The points are given as:

(5, −2) and (3, 4)

Start by calculating the slope (m)

m =(y2 - y1)/(x2 - x1)

This gives

m = (4 + 2)/(3 - 5)

Evaluate

m = -3

The equation is then calculated as:

y = m(x - x1) + y1

This gives

y = -3(x + 2) +5

So, we have:

y = -3x - 6 + 5

Evaluate the like terms

y = -3x - 1

Rewrite as:

3x + y = 1

Hence, the equation of the line is 3x + y = 1

Read more about linear equations at:

https://brainly.com/question/1884491

#SPJ1


Related Questions

Find the Slope of the line that passes through (5, -10) and (5, -4)
A. - 14/10
B. 10/14
C. 0
D. Undefined

Answers

the answer to this would be D. undefined, the slope is infinite. hope this helps!

Can you all help? Giving brainliest to whoever can help me!

Answers

Answer:

x=11

Step-by-step explanation:

15x+15=180      they are supplementary angles to each other so their sum is

                        equal to 180 degrees.

subtract 15 from both sides

15x=165

divide each side by 15

x=11

A recipe for cake calls for 3/4 cup of sugar and 2 cups of flour.

Write an equation to represent the relationship between s, sugar, and f, flour.

Answers

Answer:

32 tablespoons of sugar.

Step-by-step explanation:

Answer:

32 tablespoons of sugar.

Step-by-step explanation:

Each unit on the grid represents 5
feet. About how far, in feet, is the
drinking fountain from
the bench?
A. 8.5 feet
B. 37.1 feet
C. 33.5 feet
D. 42.7 feet

Answers

Answer:

There's no grid but I'd say A if you can give me the grid I'll edit my answer

Step-by-step explanation:

Simplify the following
(c) 2(а - 3) - 5[a - 2(a - 4)

Answers

Step-by-step explanation:

Let's simplify step-by-step.

2(a−3)−5a−2(a−4)

Distribute:

=(2)(a)+(2)(−3)+−5a+(−2)(a)+(−2)(−4)

=2a+−6+−5a+−2a+8

Combine Like Terms:

=2a+−6+−5a+−2a+8

=(2a+−5a+−2a)+(−6+8)

=−5a+2

One day 1 2 tried to make a pie. Unfortunately, she only had 3, and no sugar, so she decided to use 4 to sweeten it. The pie bubbled over in the oven and looked a mess. Not one to be discouraged, she decided to remedy the situation by covering the pie with 5 . When her 6 came over, and saw what she'd done, they 7 because 8 her pie ended up 9 but the 10 still needs to be cleaned

Answers

Answer:

even though there's no options or context,most likely answers are 1.little

2.Suzzy

3. flour

4.molases

5.foil

6 aunt

7. ate

8.

9.good

10.kitchen

A circular bracelet has a radius of
5cm

Answers

Answer:

ye

Step-by-step explanation:

..

om need hjbhhbb points bbbvvghhhhjjnnnnnn

Answer:

la cantidas es 45 mas 8 ya ñara poder tener ka cagdlfenjelsflahevveyjew

Mind lending me a hand

Answers

Answer:

54

Step-by-step explanation:

55 + 74 + 78 + z + z + 45 = 360

252 + 2z = 360

2z = 108

z = 54

Answer: 288

Step-by-step explanation:

78+74+55=207

The degree of a pentagon is 540°

so 540-207=333

so z has to be 333 but the question says +45 and that would make 333 become 378 and we need to get exactly to 333 so the answer cant be 333 but a number that 45 can be added to ,to make 333 for example 54+45=99 so 54 isnt the answer,next is 144+45 which is 189 that doesnt equall 333 so that isnt the answer either lastly is 288+45 which is 333 Thats the answer ...

Scale is 1cm = 8km. Hatboro and Smithville are 24cm apart on the map, how far apart are they?
a.87 km
b.192 km
c. 216 km
d. 33 km

Answers

Answer:

B.) 192 km

Step-by-step explanation:

Since 8km is for 1 centimeter, we multiply 8 by 24 to find how many kilometers there are for 24 centimeters. 8 * 24 is 192 so B.) is your answer.

Suppose you write a system of equations to find out which of two video stores is cheaper to join, depending on the number of videos you will rent. You solve the system and find that it has infinitely many solutions. What would that mean in terms of the problem? A. One video store is more expensive than the other no matter how many videos are rented. B. One video store is cheaper up to a certain number of videos rented, and then the other is cheaper. C. Both video stores cost the same no matter how many videos are rented. D. The problem cannot be solved.

Answers

The right answer is c

Do you guys mine lending me a hand?

Answers

Answer:the answer is 146 or 126

Step-by-step explanation:

84+112+102+96=394 and a pentagon's degrees is 540

so you have to do 540-34=146

so that is 146

the answer is 146

If you place a 13 foot ladder against the top of a 12 foot building, how many feet will the bottom of the ladder be from the bottom of the building?

Answers

Did this a while ago but I’m pretty sure it’s 5 feet x=5

11. Justin needs to rent a car to go on vacation. EnvoCar charges $65 and $0.10 per mile. FreedomRide charges $80 and $0.05 per mile.
_________________________
a. How much more does Freedom Ride charge for driving m miles than EnvoCar?
_________________________
b. If Justin’s round-trip distance is 250 miles, which car rental company should he rent from? Explain.

Answers

9514 1404 393

Answer:

  a) difference of charges is 15 -0.05m

  b) Justin should rent from EnvoCar

Step-by-step explanation:

a) EnvoCar charges ...

  c = 65 +0.10m . . . . . c = charge for m miles

FreedomRide charges ...

  c = 80 +0.05m

Then FreedomRide charges more by ...

  (80 +0.05m) -(65 +0.10m) = 15 -0.05m

__

b) If Justin's round-trip distance is 250 miles, the extra charge from FreedomRide is ...

  15 -0.05(250) = 2.50

For that distance, EnvoCar costs less. Justin should rent from EnvoCar.

_____

Additional comment

The additional charge from FreedomRide is zero when m=300. Above that mileage, FreedomRide charges less.

Which of the following represents the solution set to 4x^2+8x-1=0?

Answers

Answer:

4

Step-by-step explanation:

What is the first step in solving this equation :
2x - 7 = 19

Options :

A - Add 7 to both sides
B - Subtract 19 from both sides
C - Subtract 7 from both sides
D - Divide both sides by 2

Please help its due in 20 minutes!!​

Answers

the answer is:

2x = 19 + 7

2x= 26

x= 13

Finding a percentage of o total amount without a calculator...
An item is regularly priced at $35. It is on sale for 40% off the regular price. How much (In dollars) Is discounted from the regular price?
Amount discounted: si
х
5
?
Help

Answers

35 - 0.80*35 = x

35 - 28 = x

7 = x

x = 7

He paid $7

9 (x - 2) = 7x + 5
Does x have a value hurry only 10 minutes left!!

Answers

Answer:

x = 11.5

Step-by-step explanation:

9 (x - 2) = 7x + 5

9x - 18 = 7x + 5

    +18          +18

9x = 7x + 23

-7x   - 7x

2x = 23

---    ---

2       2

x = 11.5

 

The ratio of the measure of the sides of a triangle is 9:7:3. If the perimeter of the triangle is 266 inches, find the length of the shortest side. Help me please!!

Answers

Answer:

shortest side = 42 inches

Step-by-step explanation:

9x + 7x + 3x = 266

x = 14

3x = 42

What is the volume of the prism given below?
14
12
A. 280 units3
B. 480 units 3
C. 840 units 3
D. 420 units3

Answers

AnswerAnswer:

420

Step-by-step explanation:

The volume of the prism is 840 units³ which is correct option (C).

What is volume of prism?

The volume of the prism is defined as the measure of the space occupied within a prism . The prism is a three-dimensional shape that has base area, and height. The volume of a prism is equal to the product of base area and height of a prism.

The volume of the prism= B×H

Where, B is base area of prism, and H is height of prism.

Given that,

Length of  prism (L) = 12 centimeters

Width of prism(W) = 14 centimeters

Base area (rectangle) of prism = L × W

Base area (rectangle) of prism = 12 × 14

Base area (rectangle) of prism = 168 units²

Height of prism(H) = 5 units

The volume of the  prism =  B × H

Substitute the values in above formula,

The volume of the prism = 168×5

The volume of the prism = 840 units³

Hence, the volume of the prism is 840 units³.

Learn more about volume of the prism here:

https://brainly.com/question/11336446

#SPJ5


Determine the value of x.
A) 4.96
B) 2.90
C) 11.62
D) 0.73

Answers

The value of x in the triangle is 11.62

How to solve for x?

The complete question is in the attached image

From the image, we have the following tangent function

tan(71) = x/4

Make x the subject

x = 4 * tan(71)

Evaluate the expression

x = 11.62

Hence, the value of x is 11.62

Read more about right triangles at:

https://brainly.com/question/2437195

#SPJ1

Answer:

11.62

Step-by-step explanation:

took the test

If you graph a system of equations and they are intersect, they have no solution. True or False someone help pleaseee !!

Answers

Answer:

False

Step-by-step explanation:

If the graphs of the equations intersect, then there is one solution that is true for both equations. If the graphs of the equations do not intersect (for example, if they are parallel), then there are no solutions that are true for both equations.

I hope this is correct.

Sarah has $1000 in the bank. she took out $50 to buy supplies. What percentage of savings did she take out?

Answers

She took out 5%, 10$ is 1% so that would make 50 5%

When a number is increased by 9.5%, the result is 58. What is the original number to the nearest tenth?

(I've seen other questions like this, but the answer is wrong. It's not 50 or 52.49 or 52.5.)

Answers

Answer:

53

Step-by-step explanation:

58÷1.095= 53 is the answer

Answer:

5.03196347032

Step-by-step explanation:

We do:

.095x+x=58

1.095x=58

x=52.96

52.96 is a approximation and when you do the math and see what  9.5 of 52.96 plus 52.96 is you get 57.9912 which is super close to 8 The exact answer is x = 5.03196347032

How do I make the pain go away?

Answers

What-.....?Uhh u good?

In which of the following cases would you most likely buy a point when closing on a home loan?
a. The monthly payment is reduced by $8 and you plan to sell the home at the end of 3 years.
b. The monthly payment is reduced by $10 and you plan to sell the home at the end of 4 years.
The monthly payment is reduced by $12 and you plan to sell the home at the end of 5 years.
d. The monthly payment is reduced by $14 and you plan to sell the home at the end of 7 years.
C
Please select the best answer from the choices provided
Ο Α
C
B
0
C
OD

Answers

Answer:

d.The monthly payment is reduced by $14 and you plan to sell the home at the end of 7 years.

Step-by-step explanation:

d.The monthly payment is reduced by $14 and you plan to sell the home at the end of 7 years.Step-by-step explanation:

Given: EGEF = JK

Prove: FG = HJ

PLEASE HELP ASP

Answers

Answer:

Done.

Step-by-step explanation:

EG = EF + FG

HK = HJ + JK

We have: EG = HK and EF = JK => FG = HJ

I don't know this question. I REALLY NEED HELP!!!

Answers

Answer:

If it is asking what the answer is when you plug in 5 and 4 to the equation, it would be 54

Answer:

54

Step-by-step explanation:

10m + [tex]\frac{n^{2} }{4}[/tex]

First insert the m value and then multiply

10m = 10(5) =50

then the equation becomes 50 +  [tex]\frac{n^{2} }{4}[/tex]  

Second insert the n value

[tex]\frac{4^{2} }{4}[/tex]

4 to the second power = 16 so it becomes 16 over 4 aka 4 wholes

50+16/4

50+4=54

A store has a 20% off sale on pants. With this discount, the price of one pair of paints is $15.20. What was the original price of the pants?

Answers

$19. 19 x 0.8 = 15.2

a^2 + b^2 + 2bc - 2ca - 2ab

Answers

I got a^2 + b^2+2bc-2ac-2ab

If a varies directly as the cube root of b and if a=3 and b=64.find the formula connecting the variables hence find b when a=15/4

Answers

Answer:

b = 125

Step-by-step explanation:

Given a varies directly as [tex]\sqrt[3]{b}[/tex] then the equation relating them is

a = k[tex]\sqrt[3]{b}[/tex] ← k is the constant of variation

To find k use the condition a = 3 , b = 64 , then

3 = k[tex]\sqrt[3]{64}[/tex] = 4k ( divide both sides by 4 )

[tex]\frac{3}{4}[/tex] = k

a = [tex]\frac{3}{4}[/tex] [tex]\sqrt[3]{b}[/tex] ← equation of variation

When a = [tex]\frac{15}{4}[/tex] , then

[tex]\frac{15}{4}[/tex] = [tex]\frac{3}{4}[/tex] [tex]\sqrt[3]{b}[/tex] ( multiply both sides by 4 to clear the fractions )

15 = 3[tex]\sqrt[3]{b}[/tex] ( divide both sides by 3 )

5 = [tex]\sqrt[3]{b}[/tex] , then

b = 5³ = 125

Other Questions
Assume we want to execute the DAXPY loop show on page 511 in MIPS assembly on the NVIDIA 8800 GTX GPU described in this chapter. In this problem, we will assume that all math operations are performed on single-precision floating-point numbers (we will rename the loop SAXPY). Assume that instructions take the following number of cycles to execute.[20] Describe how you will constructs warps for the SAXPY loop to exploit the 8 cores provided in a single multiprocessor when you look at the screen rather than your camera while presenting online you appear to proponents of overturning roe v. wade are popularly and collectively referred to as the A sealed rigid vessel contains air at STP. It is heated to bring the air to a temperature of 80 C.What will be the ratio of the mean free path of the air molecules at 80 C to their mean free path at STP? Breast cancer is the number 1 cause of cancer death among young women. Which of the followingcriteria for an ideal screening program is best illustrated by this statement?A: Substantial mortality and/or morbidityB: Early detection improves outcomeC: Screening is feasibleD: Screening is acceptable in terms of costs, harms and patient acceptanceE: None of the above discuss the conformity costs faced by a politician who has adopted a party label. When policymakers make decisions in response to a pre-specified rule, they are undertaking A) active policy B) discretionary policy. C) passive policy. D) irrational policy Show that the total ground-state energy of N fermions in a three-dimensional box is given by R_total = 3/5 N E_F Thus the average energy per fermion is 3E_F/5 Use the following list of accounts for Milner's Star Express Cleaning Service. Cash $2,026 Fees Earned 13,835 Accounts Payable 7,530 D. Milner, Capital January 1, 20-- 6,000 D. Milner, Drawing 1,750 Utilities Expense 153 Prepaid Insurance 1,216 Rent Expense 1,200 Accounts Receivable 4,080 Equipment 15,290 Wages Expense 1,650 Required: 1. Prepare an income statement for the year ended December 31, 20 draw the structure of a triglyceride that contains one myristic acid, one palmitoleic acid, and one linoleic acid. _________is a large phagocytic cell that has a high capacity for killing microbes and cleaning Calculate the pH of a buffer that contains 1. 00 M NH3 and 0. 75 M NH4Cl. The Kb value for NH3 is 1. 8 10-5 use equation i=r2dmi=r2dm to calculate the moment of inertia of a slender, uniform rod with mass mm and length ll about an axis at one end, perpendicular to the rod. a lot of 30 watches is 20 efective. what is the probability that a sample of 3 will contain 2 defectives TRUE/FALSE. (T/F) A reportable incident is any event that could lead to a lawsuit. As a soldier, activity is essential to your performance. Your initial recommended Activity Plus Targets are:a. Get at least 10,000 steps per dayb. Get at least 150 minutes of moderate aerobic exercise per weekc. Get at least 2 days or more of a resistance training per week milk comes in opaque containers because _____ is destroyed by exposure to light. if you add a competitive inhibitor of enzyme e1 to a cell, which species would increase in concentration in the cell? 1. The heat of vaporization of water is 540 cal/g, and the heat of fusion is 80 cal/g. The heat capacity of liquid water is 1 cal g-1c-1, and the heat capacity of ice is 0.5 cal g-1 c-1. What amount of heat is required to evaporate 20 g of water at 100 C. ___ cal . 2. 28 g of ice at -10c is heated until it becomes liquid water at 28c. how much heat was required for this to occur? ___ cal help plsssssssssssssssss